Source code for qrisp.core.library

"""
\********************************************************************************
* Copyright (c) 2023 the Qrisp authors
*
* This program and the accompanying materials are made available under the
* terms of the Eclipse Public License 2.0 which is available at
* http://www.eclipse.org/legal/epl-2.0.
*
* This Source Code may also be made available under the following Secondary
* Licenses when the conditions for such availability set forth in the Eclipse
* Public License, v. 2.0 are satisfied: GNU General Public License, version 2
* with the GNU Classpath Exception which is
* available at https://www.gnu.org/software/classpath/license.html.
*
* SPDX-License-Identifier: EPL-2.0 OR GPL-2.0 WITH Classpath-exception-2.0
********************************************************************************/
"""


import numpy as np

import qrisp.circuit.standard_operations as std_ops
from qrisp.core import recursive_qs_search

def append_operation(operation, qubits=[], clbits=[]):
    from qrisp import find_qs
    
    qs = find_qs(qubits)
    
    qs.append(operation, qubits, clbits)


[docs]def cx(control, target): """ Applies a CX gate. Parameters ---------- control : Qubit or list[Qubit] or QuantumVariable The Qubit to control on. target : Qubit or list[Qubit] or QuantumVariable The Qubit to perform the X gate on. """ append_operation(std_ops.CXGate(), [control, target]) # std_ops.CXGate().append([qubits_0, qubits_1]) return control, target
[docs]def cy(control, target): """ Applies a CY gate. Parameters ---------- control : Qubit or list[Qubit] or QuantumVariable The Qubit to control on. target : Qubit or list[Qubit] or QuantumVariable The Qubit to perform the Y gate on. """ append_operation(std_ops.CYGate(), [control, target]) return control, target
[docs]def cz(control, target): """ Applies a CZ gate. Parameters ---------- control : Qubit or list[Qubit] or QuantumVariable The Qubit to control on. target : Qubit or list[Qubit] or QuantumVariable The Qubit to perform the Z gate on. """ append_operation(std_ops.CZGate(), [control, target]) return control, target
[docs]def h(qubits): """ Applies an H gate. Parameters ---------- qubits : Qubit or list[Qubit] or QuantumVariable The Qubit to perform the H gate on. """ append_operation(std_ops.HGate(), [qubits]) return qubits
[docs]def x(qubits): """ Applies an X gate. Parameters ---------- qubits : Qubit or list[Qubit] or QuantumVariable The Qubit to perform the X gate on. """ append_operation(std_ops.XGate(), [qubits]) return qubits
[docs]def y(qubits): """ Applies a Y gate. Parameters ---------- qubits : Qubit or list[Qubit] or QuantumVariable The Qubit to perform the Y gate on. """ append_operation(std_ops.YGate(), [qubits]) return qubits
[docs]def z(qubits): """ Applies a Z gate. Parameters ---------- qubits : Qubit or list[Qubit] or QuantumVariable The Qubit to perform the Z gate on. """ append_operation(std_ops.ZGate(), [qubits]) return qubits
[docs]def mcx(controls, target, method="auto", ctrl_state=-1, num_ancilla=1): r""" Applies a multi-controlled X gate. The following methods are available: .. list-table:: :widths: 20 80 :header-rows: 1 * - Method - Description * - ``gray`` - Performs a gray code traversal which requires no ancillae but is rather inefficient for large numbers of control qubits. * - ``gray_pt``/``gray_pt_inv`` - More efficient but introduce extra phases that need to be uncomputed by performing the inverse of this gate on the same inputs. For more information on phase tolerance, check `this paper <https://iopscience.iop.org/article/10.1088/2058-9565/acaf9d/meta>`_. * - ``balauca`` - Method based on this `paper <https://www.iccs-meeting.org/archive/iccs2022/papers/133530169.pdf>`_ with logarithmic depth but requires many ancilla qubits. * - ``maslov`` - Documented `here <https://arxiv.org/abs/1508.03273>`_, requires less ancilla qubits but is only available for 4 or less control qubits. * - ``yong`` - Can be found int this `article <https://link.springer.com/article/10.1007/s10773-017-3389-4>`_.This method requires only a single ancilla and has moderate scaling in depth and gate count. * - ``amy`` - A Toffoli-circuit (ie. only two control qubits are possible), which (temporarily) requires one ancilla qubit. However, instead of the no-ancilla T-depth 4, this circuit achieves a T-depth of 2. Find the implementation details in `this paper <https://arxiv.org/pdf/1206.0758.pdf>`_. * - ``jones`` - Similar to ``amy`` but uses two ancilla qubits, and has a T-depth of 1. Read about it `here <https://arxiv.org/abs/1212.5069>`_. * - ``gidney`` - A very unique way for synthesizing a logical AND. The Gidney Logical AND performs a circuit with T-depth 1 to compute the truth value and performs another circuit involving a measurement and a classically controlled CZ gate for uncomputation. The uncomputation circuit has T-depth 0, such that the combined T-depth is 1. Requires no ancillae. More details `here <https://arxiv.org/abs/1709.06648>`_. Works only for two control qubits. * - ``hybrid`` - A flexible method which combines the other available methods, such that the amount of used ancillae is customizable. After several ``balauca``-layers, the recursion is canceled by either a ``yong``, ``maslov`` or ``gray`` mcx, depending on what fits the most. * - ``auto`` - Recompiles the mcx gate at compile time using the hybrid algorithm together with the information about how many clean/dirty ancillae qubits are available. For more information check :meth:`qrisp.QuantumSession.compile`. .. note:: Due to Qrisp's automatic qubit management, clean ancilla qubits are not as much of a sparse resource as one might think. Even though the ``balauca`` method requires a considerable amount of ancillae, many other functions also do, implying there is alot of recycling potential. The net effect is that in more complex programs, the amount of qubits of the circuit returned by the :meth:`compile method <qrisp.QuantumSession.compile>` increases only slightly. Parameters ---------- controls : list[Qubits] or QuantumVariable The Qubits to control on. target : Qubit The Qubit to perform the X gate on. method : str, optional The synthesis method. Available are ``auto``, ``gray``, ``gray_pt``, ``gray_pt_inv``, ``maslov``, ``balauca`` and ``yong``. The default is ``auto``. ctrl_state : int or str, optional The state on which to activate the X gate. The default is "1111..". num_ancilla : int, optional Specifies the amount of ancilla qubits to use. This parameter is used only if the method is set to ``hybrid``. The default is 1. Examples -------- We apply a 3-contolled X gate >>> from qrisp import QuantumVariable, mcx >>> control = QuantumVariable(3) >>> target = QuantumVariable(1) >>> mcx(control, target, method = "gray") >>> print(control.qs) :: QuantumCircuit: -------------- control.0: ──■── control.1: ──■── control.2: ──■── ┌─┴─┐ target.0: ┤ X ├ └───┘ Live QuantumVariables: --------------------- QuantumVariable control QuantumVariable target We compare different performance indicators. :: from qrisp import QuantumVariable, mcx def benchmark_mcx(n, methods): for method in methods: controls = QuantumVariable(n) target = QuantumVariable(1) mcx(controls, target, method = method) compiled_qc = controls.qs.compile() print(f"==================\nMethod: {method}\n------------------") print(f"Depth: \t\t\t{compiled_qc.depth()}") print(f"CNOT count: \t{compiled_qc.cnot_count()}") print(f"Qubit count: \t{compiled_qc.num_qubits()}") >>> benchmark_mcx(4, methods = ["gray", "gray_pt", "maslov", "balauca", "yong"]) ================== Method: gray ------------------ Depth: 50 CNOT count: 30 Qubit count: 5 ================== Method: gray_pt ------------------ Depth: 34 CNOT count: 16 Qubit count: 5 ================== Method: maslov ------------------ Depth: 43 CNOT count: 18 Qubit count: 6 ================== Method: balauca ------------------ Depth: 22 CNOT count: 18 Qubit count: 7 ================== Method: yong ------------------ Depth: 77 CNOT count: 30 Qubit count: 6 >>> benchmark_mcx(10, methods = ["gray", "gray_pt", "balauca", "yong"]) ================== Method: gray ------------------ Depth: 3106 CNOT count: 2046 Qubit count: 11 ================== Method: gray_pt ------------------ Depth: 2050 CNOT count: 1024 Qubit count: 11 ================== Method: balauca ------------------ Depth: 53 CNOT count: 54 Qubit count: 18 ================== Method: yong ------------------ Depth: 621 CNOT count: 264 Qubit count: 12 **Mid circuit measurement based methods** The ``gidney`` and ``jones`` method are unique in the way that they require mid circuit measurements. The measurements are inserted retroactively by the :meth:`.compile <qrisp.QuantumSession.compile>` method, because immediate compilation would prevent evaluation of the statevector (since a measurement is involved). Instead a tentative (measurement free) representative is inserted and replaced at compile time. To get a better understanding consider the following script: >>> from qrisp import QuantumVariable, mcx >>> control = QuantumVariable(2) >>> target = QuantumVariable(1) >>> mcx(control, target, method = "jones") >>> print(control.qs) QuantumCircuit: --------------- ┌───────────────────────────┐ control.0: ┤0 ├ │ │ control.1: ┤1 ├ │ │ target.0: ┤2 uncompiled_jones_toffoli ├ │ │ jones_ancilla.0: ┤3 ├ │ │ jones_ancilla.1: ┤4 ├ └───────────────────────────┘ Live QuantumVariables: ---------------------- QuantumVariable control QuantumVariable target We see that there is no classical bit and therefore also no measurement. The statevector can still be accessed: >>> print(control.qs.statevector()) |00>*|0> To introduce the measurement we simply call the :meth:`.compile <qrisp.QuantumSession.compile>` method with the keyword ``compile_mcm = True``: >>> qc = control.qs.compile(compile_mcm = True) >>> print(qc) ┌─────────────────────────┐ control.0: ┤0 ├ │ │ control.1: ┤1 ├ │ │ target.0: ┤2 ├ │ compiled_jones_toffoli │ workspace_0: ┤3 ├ │ │ workspace_1: ┤4 ├ │ │ cb_0: ╡0 ╞ └─────────────────────────┘ Because there is a measurement now, the statevector can no longer be accessed. >>> qc.statevector_array() Exception: Unitary of operation measure not defined. However the T-depth went down by 50%: >>> print(qc.t_depth()) 1 >>> print(control.qs.compile(compile_mcm = False).t_depth()) 2 A similar construction holds for the `Gidney's temporary logical AND <https://arxiv.org/abs/1709.06648>`_. However there are additional details: This technique always comes in pairs. A computation and an uncomputation. The computation circuit has a T-depth of 1 and the uncomputation circuit has a T-depth of 0. The uncomputation circuit however contains a measurement. As you can imagine, this measurement is also inserted at compile time. Even though both circuits are not the inverses of each other, Qrisp will use the respective partner if called to invert: >>> control = QuantumVariable(2) >>> target = QuantumVariable(1) >>> mcx(control, target, method = "gidney") >>> print(control.qs) QuantumCircuit: --------------- ┌────────────────────────┐ control.0: ┤0 ├ │ │ control.1: ┤1 uncompiled_gidney_mcx ├ │ │ target.0: ┤2 ├ └────────────────────────┘ Live QuantumVariables: ---------------------- QuantumVariable control QuantumVariable target This even works in conjunction with the :ref:`uncomputation module <Uncomputation>`: >>> target.uncompute() >>> print(target.qs) QuantumCircuit: --------------- ┌────────────────────────┐┌────────────────────────────┐ control.0: ┤0 ├┤0 ├ │ ││ │ control.1: ┤1 uncompiled_gidney_mcx ├┤1 uncompiled_gidney_mcx_inv ├ │ ││ │ target.0: ┤2 ├┤2 ├ └────────────────────────┘└────────────────────────────┘ Live QuantumVariables: ---------------------- QuantumVariable control To introduce the measurement, we call the compile method. >>> print(target.qs.compile(compile_mcm = True)) ┌──────────────────────┐┌──────────────────────────┐ control.0: ┤0 ├┤0 ├ │ ││ │ control.1: ┤1 compiled_gidney_mcx ├┤1 ├ │ ││ compiled_gidney_mcx_inv │ workspace_0: ┤2 ├┤2 ├ └──────────────────────┘│ │ cb_0: ════════════════════════╡0 ╞ └──────────────────────────┘ Apart from uncomputation, the inverted Gidney mcx can also be accessed via, the :ref:`InversionEnvironment`: :: from qrisp import invert control = QuantumVariable(2) target = QuantumVariable(1) with invert(): mcx(control, target, method = "gidney") >>> print(control.qs) QuantumCircuit: --------------- ┌────────────────────────────┐ control.0: ┤0 ├ │ │ control.1: ┤1 uncompiled_gidney_mcx_inv ├ │ │ target.0: ┤2 ├ └────────────────────────────┘ Live QuantumVariables: ---------------------- QuantumVariable control QuantumVariable target """ from qrisp.misc import bin_rep from qrisp.mcx_algs import GidneyLogicalAND, amy_toffoli, jones_toffoli from qrisp.core import QuantumVariable from qrisp.qtypes import QuantumBool new_controls = [] for qbl in controls: if isinstance(qbl, QuantumBool): new_controls.append(qbl[0]) else: new_controls.append(qbl) if isinstance(target, (list, QuantumVariable)): if len(target) > 1: raise Exception("Target of mcx contained more than one qubit") target = target[0] qubits_0 = new_controls qubits_1 = [target] n = len(qubits_0) if n == 0: return controls, target if not isinstance(ctrl_state, str): if ctrl_state == -1: ctrl_state += 2**n ctrl_state = bin_rep(ctrl_state, n)[::-1] if len(ctrl_state) != n: raise Exception( f"Given control state {ctrl_state} does not match control qubit amount {n}" ) from qrisp.mcx_algs import ( balauca_dirty, balauca_mcx, hybrid_mcx, maslov_mcx, yong_mcx, ) if method in ["gray", "gray_pt", "gray_pt_inv"] or len(qubits_0) == 1: if len(qubits_0) == 1: method = "gray" append_operation( std_ops.MCXGate(len(qubits_0), ctrl_state, method=method), qubits_0 + qubits_1, ) elif method == "gidney": if len(qubits_0) != 2: raise Exception(f"Tried to call Gidney logical AND with {len(qubits_0)} controls instead of two") append_operation( GidneyLogicalAND(ctrl_state = ctrl_state), qubits_0 + qubits_1, ) elif method == "gidney_inv": if len(qubits_0) != 2: raise Exception(f"Tried to call Gidney logical AND with {len(qubits_0)} controls instead of two") append_operation( GidneyLogicalAND(ctrl_state = ctrl_state, inv = True), qubits_0 + qubits_1, ) elif method == "maslov": from qrisp import QuantumBool if n >= 3: ancilla = [QuantumBool(name="maslov_anc_")] else: ancilla = [] append_operation(maslov_mcx(n, ctrl_state), qubits_0 + ancilla + qubits_1) [qv.delete() for qv in ancilla] elif method == "balauca": balauca_mcx(qubits_0, qubits_1, ctrl_state=ctrl_state) elif method == "balauca_dirty": balauca_dirty(qubits_0, qubits_1, k=num_ancilla, ctrl_state=ctrl_state) elif method == "yong": yong_mcx(qubits_0, qubits_1, ctrl_state=ctrl_state) elif method == "hybrid": hybrid_mcx(qubits_0, qubits_1, ctrl_state=ctrl_state, num_ancilla=num_ancilla) elif method == "amy": if len(qubits_0) != 2: raise Exception(f"Tried to call Amy MCX with {len(qubits_0)} controls instead of two") amy_toffoli(qubits_0, qubits_1, ctrl_state = ctrl_state) elif method == "jones": if len(qubits_0) != 2: raise Exception(f"Tried to call Jones MCX with {len(qubits_0)} controls instead of two") jones_toffoli(qubits_0, qubits_1, ctrl_state = ctrl_state) elif method == "auto": # if n <= 3: # return mcx(qubits_0, qubits_1, method = "gray", ctrl_state = ctrl_state) # if 3 < n < 5: # return mcx(qubits_0, qubits_1, method = "maslov", ctrl_state = ctrl_state) # else: # return mcx(qubits_0, qubits_1, method = "balauca", ctrl_state = ctrl_state) # noqa:501 gate = std_ops.MCXGate(len(qubits_0), ctrl_state, method="auto") append_operation(gate, qubits_0 + qubits_1) return controls, target
[docs]def mcz(qubits, method="auto", ctrl_state=-1, num_ancilla=1): """ Applies a multi-controlled Z gate. For more information on the available methods, check :meth:`the mcx documentation page <qrisp.mcx>`. Parameters ---------- qubits : QuantumVariable or list[Qubits] The Qubits to control on. method : str, optional The synthesis method. Available are ``auto``, ``gray``, ``gray_pt``, ``gray_pt_inv``, ``maslov``, ``balauca``, ``yong`` and ``hybrid``. The default is ``auto``. ctrl_state : int or str, optional The state on which to activate the Z gate. The default is "1111...". num_ancilla : int, optional Specifies the amount of ancilla qubits to use. This parameter is used only if the method is set to ``hybrid``. The default is 1. """ from qrisp.misc import gate_wrap @gate_wrap(permeability="full", is_qfree=True, name="anc supported mcz") def mcz_inner(qubits, method="auto", ctrl_state=-1): if len(ctrl_state) != n: raise Exception( f"Given control state {ctrl_state} does not match" f"control qubit amount {n}" ) from qrisp import h, x if ctrl_state[-1] == "0": x(qubits[-1]) h(qubits[-1]) mcx(qubits[:-1], qubits[-1], method=method, ctrl_state=ctrl_state[:-1]) h(qubits[-1]) if ctrl_state[-1] == "0": x(qubits[-1]) return qubits n = len(qubits) from qrisp import bin_rep if not isinstance(ctrl_state, str): if ctrl_state == -1: ctrl_state += 2**n ctrl_state = bin_rep(ctrl_state, n) if method in ["gray", "auto"]: if ctrl_state[-1] == "0": x(qubits[-1]) if len(qubits) > 1: append_operation( std_ops.ZGate().control( len(qubits) - 1, method=method, ctrl_state=ctrl_state[:-1] ), qubits, ) else: z(qubits[0]) if ctrl_state[-1] == "0": x(qubits[-1]) return qubits return mcz_inner(qubits, method, ctrl_state)
[docs]def mcp(phi, qubits, method="auto", ctrl_state=-1): """ Applies a multi-controlled phase gate. The available methods are: * ``gray`` , which performs a traversal of the gray code. * ``balauca`` , which is a modified version of the algorithm presented `here <https://www.iccs-meeting.org/archive/iccs2022/papers/133530169.pdf>`_. * ``auto`` , which picks ``gray`` for any qubit count less than 4 and ``balauca`` otherwise. Parameters ---------- phi : float or sympy.Symbol The phase to apply. qubits : list[Qubit] or QuantumVariable The qubits to apply the multi-controlled phase gate on. method : str, optional The method to deploy. The default is "auto". ctrl_state : str or int, optional The control state on which to apply the phase. The default is "111...". """ from qrisp.mcx_algs import balauca_mcx from qrisp.misc import bin_rep, gate_wrap @gate_wrap(permeability="full", is_qfree=True, name="anc supported mcp") def balauca_mcp(phi, qubits, ctrl_state): from qrisp.circuit.quantum_circuit import convert_to_qb_list qubits = convert_to_qb_list(qubits) if ctrl_state[-1] == "0": x(qubits[-1]) balauca_mcx(qubits[:-1], [qubits[-1]], ctrl_state=ctrl_state, phase=phi) if ctrl_state[-1] == "0": x(qubits[-1]) n = len(qubits) if not isinstance(ctrl_state, str): if ctrl_state == -1: ctrl_state += 2**n ctrl_state = bin_rep(ctrl_state, n)[::-1] n = len(qubits) if method == "gray" or method == "gray_pt": if ctrl_state[-1] == "0": x(qubits[-1]) append_operation( std_ops.PGate(phi).control(n - 1, ctrl_state=ctrl_state[:-1], method = method), qubits ) if ctrl_state[-1] == "0": x(qubits[-1]) return qubits elif method == "balauca": balauca_mcp(phi, qubits, ctrl_state=ctrl_state) return qubits elif method == "auto": if n < 4: return mcp(phi, qubits, method="gray", ctrl_state=ctrl_state) else: return mcp(phi, qubits, method="balauca", ctrl_state=ctrl_state) else: raise Exception(f"Don't know method {method}")
[docs]def p(phi, qubits): """ Applies a phase gate. Parameters ---------- phi : float or sympy.Symbol The phase to apply. qubits : Qubit or list[Qubit] or QuantumVariable The Qubit on which to apply the phase gate. """ append_operation(std_ops.PGate(phi), [qubits]) # std_ops.PGate(phi).append([qubits]) return qubits
[docs]def cp(phi, qubits_0, qubits_1): """ Applies a controlled phase gate. Parameters ---------- phi : float or sympy.Symbol The phase to apply. qubits_0 : Qubit or list[Qubit] or QuantumVariable The first Qubit. qubits_1 : Qubit or list[Qubit] or QuantumVariable The second Qubit. """ append_operation(std_ops.CPGate(phi), [qubits_0, qubits_1]) # std_ops.CPGate(phi).append([qubits_0, qubits_1]) return qubits_0, qubits_1
[docs]def rx(phi, qubits): """ Applies an RX gate. Parameters ---------- phi : float or sympy.Symbol The angle parameter. qubits : Qubit or list[Qubit] or QuantumVariable The Qubit to perform the RX gate on. """ append_operation(std_ops.RXGate(phi), [qubits]) return qubits
[docs]def ry(phi, qubits): """ Applies an RY gate. Parameters ---------- phi : float or sympy.Symbol The angle parameter. qubits : Qubit or list[Qubit] or QuantumVariable The Qubit to perform the RY gate on. """ append_operation(std_ops.RYGate(phi), [qubits]) return qubits
[docs]def rz(phi, qubits): """ Applies an RY gate. Parameters ---------- phi : float or sympy.Symbol The angle parameter. qubits : Qubit or list[Qubit] or QuantumVariable The Qubit to perform the RY gate on. """ append_operation(std_ops.RZGate(phi), [qubits]) return qubits
[docs]def crz(phi, qubits_0, qubits_1): """ Applies controled RZ gate Parameters ---------- phi : float or sympy.Symbol The angle parameter. qubits_0 : Qubit or list[Qubit] or QuantumVariable The first Qubit. qubits_1 : Qubit or list[Qubit] or QuantumVariable The second Qubit. """ append_operation(std_ops.RZGate(phi).control(1), [qubits_0, qubits_1]) return qubits_0, qubits_1
[docs]def s(qubits): """ Applies an S gate. Parameters ---------- qubits : Qubit or list[Qubit] or QuantumVariable The Qubit to perform the S gate on. """ append_operation(std_ops.SGate(), [qubits]) return qubits
[docs]def t(qubits): """ Applies a T gate. Parameters ---------- qubits : Qubit or list[Qubit] or QuantumVariable The Qubit to perform the T gate on. """ append_operation(std_ops.TGate(), [qubits]) return qubits
[docs]def s_dg(qubits): """ Applies a daggered S gate. Parameters ---------- qubits : Qubit or list[Qubit] or QuantumVariable The Qubit to perform the daggered S gate on. """ append_operation(std_ops.SGate().inverse(), [qubits]) return qubits
[docs]def t_dg(qubits): """ Applies a daggered T gate. Parameters ---------- qubits : Qubit or list[Qubit] or QuantumVariable The Qubit to perform the daggered T gate on. """ append_operation(std_ops.TGate().inverse(), [qubits]) return qubits
[docs]def sx(qubits): """ Applies an SX gate. Parameters ---------- qubits : Qubit or list[Qubit] or QuantumVariable The Qubit to perform the SX gate on. """ append_operation(std_ops.SXGate().inverse(), [qubits]) return qubits
[docs]def sx_dg(qubits): """ Applies a daggered SX gate. Parameters ---------- qubits : Qubit or list[Qubit] or QuantumVariable The Qubit to perform the daggered SX gate on. """ append_operation(std_ops.SXDGGate().inverse(), [qubits]) return qubits
[docs]def gphase(phi, qubits): """ Applies a global phase. This gate turns into a phase gate when controlled. Parameters ---------- phi : float or sympy.Symbol The global phase to apply. qubits : Qubit or list[Qubit] or QuantumVariable The Qubit to perform the global phase gate on. """ append_operation(std_ops.GPhaseGate(phi), qubits) return qubits
[docs]def xxyy(phi, beta, qubits_0, qubits_1): """ Applies an XXYY interaction gate. Parameters ---------- phi : float or sympy.Symbol The global phase to apply. beta : float or sympy.Symbol The other angle parameter. qubits_0 : Qubit or list[Qubit] or QuantumVariable The first Qubit to perform the XXYY gate on. qubits_1 : Qubit or list[Qubit] or QuantumVariable The second Qubit to perform the XXYY gate on. """ append_operation(std_ops.XXYYGate(phi, beta), [qubits_0, qubits_1]) return qubits_0, qubits_1
[docs]def rzz(phi, qubits_0, qubits_1): """ Applies an RZZ gate. Parameters ---------- phi : float or sympy.Symbol The phase to apply. beta : float or sympy.Symbol The other angle parameter. qubits_0 : Qubit or list[Qubit] or QuantumVariable The first argument to perform the RZZ gate one. qubits_1 : Qubit or list[Qubit] or QuantumVariable The second argument to perform the RZZ gate one. """ append_operation(std_ops.RZZGate(phi), [qubits_0, qubits_1]) return qubits_0, qubits_1
[docs]def rxx(phi, qubits_0, qubits_1): """ Applies an RXX gate. Parameters ---------- phi : float or sympy.Symbol The phase to apply. beta : float or sympy.Symbol The other angle parameter. qubits_0 : Qubit or list[Qubit] or QuantumVariable The first argument to perform the RXX gate one. qubits_1 : Qubit or list[Qubit] or QuantumVariable The second argument to perform the RXX gate one. """ append_operation(std_ops.RXXGate(phi), [qubits_0, qubits_1]) return qubits_0, qubits_1
[docs]def u3(theta, phi, lam, qubits): """ Applies an U3 gate. Parameters ---------- theta : float or sympy.Symbol The first angle parameter. phi : float or sympy.Symbol The second angle parameter. lambda : float or sympy.Symbol The third angle parameter. qubits : Qubit or list[Qubit] or QuantumVariable The Qubit to perform the U3 gate on. """ append_operation(std_ops.RZGate(phi), [qubits]) return qubits
[docs]def measure(qubits, clbits=None): """ Performs a measurement of the specified Qubit. Parameters ---------- qubit : Qubit or list[Qubit] or QuantumVariable The Qubit to measure. clbit : Clbit, optional The Clbit to store the result in. By default, a new Clbit will be created. """ if clbits is None: clbits = [] if hasattr(qubits, "__len__"): for qb in qubits: try: clbits.append(qubits[0].qs.add_clbit()) except AttributeError: clbits.append(qubits[0].qs().add_clbit()) else: clbits = qubits.qs().add_clbit() append_operation(std_ops.Measurement(), [qubits], [clbits]) return qubits
[docs]def barrier(qubits): """ A visual marker for structuring the QuantumCircuit. Parameters ---------- qubits : Qubit or list[Qubit] or QuantumVariable The Qubit to apply the barrier to. Examples -------- >>> from qrisp import QuantumVariable, x, y, barrier >>> qv = QuantumVariable(5) >>> x(qv) >>> barrier(qv) >>> y(qv) >>> print(qv.qs) :: QuantumCircuit: -------------- ┌───┐ ░ ┌───┐ qv.0: ┤ X ├─░─┤ Y ├ ├───┤ ░ ├───┤ qv.1: ┤ X ├─░─┤ Y ├ ├───┤ ░ ├───┤ qv.2: ┤ X ├─░─┤ Y ├ ├───┤ ░ ├───┤ qv.3: ┤ X ├─░─┤ Y ├ ├───┤ ░ ├───┤ qv.4: ┤ X ├─░─┤ Y ├ └───┘ ░ └───┘ Live QuantumVariables: --------------------- QuantumVariable qv """ from qrisp import Qubit if isinstance(qubits, Qubit): qubits = [qubits] append_operation(std_ops.Barrier(len(qubits)), qubits) return qubits
[docs]def swap(qubits_0, qubits_1): """ Applies a SWAP gate. Parameters ---------- qubits_0 : Qubit or list[Qubit] or QuantumVariable The first Qubit. qubits_1 : Qubit or list[Qubit] or QuantumVariable The second Qubit. """ append_operation(std_ops.SwapGate(), [qubits_0, qubits_1]) return qubits_0, qubits_1
[docs]def id(qubits): """ Applies an ID gate. Parameters ---------- qubits : Qubit or list[Qubit] or QuantumVariable The qubits to perform the ID gate on. """ append_operation(std_ops.IDGate(), [qubits]) return qubits
def QFT_inner(qv, exec_swap=True, qiskit_endian=True, inplace_mult=1, use_gms=False, inpl_adder = None): from qrisp.misc import is_inv qv = list(qv) n = len(qv) if qiskit_endian: qv = qv[::-1] if not use_gms: from qrisp.environments.quantum_environments import QuantumEnvironment env = QuantumEnvironment else: from qrisp.environments.GMS_environment import GMSEnvironment env = GMSEnvironment if not is_inv(inplace_mult, n): raise Exception( "Tried to perform non-invertible inplace multiplication" "during Fourier-Transform" ) if inpl_adder is None: accumulated_phases = np.zeros(n) for i in range(n): if accumulated_phases[i] and not use_gms: p(accumulated_phases[i], qv[i]) accumulated_phases[i] = 0 h(qv[i]) if i == n - 1: break with env(): for k in range(n - i - 1): # cp(inplace_mult * 2 * np.pi / 2 ** (k + 2), qv[k + i + 1], qv[i]) if use_gms: cp(inplace_mult * 2 * np.pi / 2 ** (k + 2), qv[i], qv[k + i + 1]) else: phase = inplace_mult * 2 * np.pi / 2 ** (k + 2) # cx(qv[k + i + 1], qv[i]) # p(-phase/2, qv[i]) # cx(qv[k + i + 1], qv[i]) cx(qv[i], qv[k + i + 1]) p(-phase/2, qv[k + i + 1]) cx(qv[i], qv[k + i + 1]) accumulated_phases[i] += phase/2 accumulated_phases[k + i + 1] += phase/2 for i in range(n): if accumulated_phases[i] and not use_gms: p(accumulated_phases[i], qv[i]) accumulated_phases[i] = 0 else: from qrisp import QuantumFloat, conjugate reservoir = QuantumFloat(n+1) def prepare_reservoir(reservoir): n = len(reservoir) h(reservoir) for i in range(n): p(np.pi*2**(i-n+1), reservoir[i]) with conjugate(prepare_reservoir)(reservoir): for i in range(n): h(qv[i]) if i == n - 1: break phase_qubits = [] for k in range(n - i - 1): cx(qv[i], qv[k + i + 1]) phase_qubits.append(qv[k + i + 1]) inpl_adder(phase_qubits[::-1], reservoir[-len(phase_qubits)-2:]) for k in range(n - i - 1): cx(qv[i], qv[k + i + 1]) x(reservoir) inpl_adder(phase_qubits[::-1], reservoir[-len(phase_qubits)-2:]) x(reservoir) s(qv) inpl_adder(qv, reservoir[-n-1:]) reservoir.delete() if exec_swap: for i in range(n // 2): swap(qv[i], qv[n - i - 1]) return qv
[docs]def QFT( qv, inv=False, exec_swap=True, qiskit_endian=True, inplace_mult=1, use_gms=False, inpl_adder=None ): """ Performs the quantum fourier transform on the input. Parameters ---------- qv : QuantumVariable QuantumVariable to transform (in-place). inv : bool, optional If set to True, the inverse transform will be applied. The default is False. exec_swap : bool, optional If set to False, the swaps at the end of the transformation will be skipped. The default is True. qiskit_endian : bool, optional If set to False the order of bits will be reversed. The default is True. inplace_mult : int, optional Allows multiplying the QuantumVariable with an extra factor during the transformation. For more information check `the publication <https://ieeexplore.ieee.org/document/9815035>`_. The default is 1. use_gms : bool, optional If set to True, the QFT will be compiled using only GMS gates as entangling gates. The default is False. inpl_adder : callable, optional Uses an adder and a reservoir state to perform the QFT. Read more about it :ref:`here <adder_based_qft>`. The default is None """ from qrisp import gate_wrap, invert name = "QFT" if not exec_swap: name += " no swap" if inplace_mult != 1: name += " inpl mult " + str(inplace_mult) if inpl_adder is not None: name += "_adder_supported" if inv: with invert(): gate_wrap(permeability=[], is_qfree=False, name=name)(QFT_inner)( qv, exec_swap=exec_swap, qiskit_endian=qiskit_endian, inplace_mult=inplace_mult, use_gms=use_gms, inpl_adder=inpl_adder ) else: gate_wrap(permeability=[], is_qfree=False, name=name)(QFT_inner)( qv, exec_swap=exec_swap, qiskit_endian=qiskit_endian, inplace_mult=inplace_mult, use_gms=use_gms, inpl_adder=inpl_adder ) return qv
[docs]def QPE( args, U, precision=None, target=None, iter_spec=False, ctrl_method=None, kwargs={} ): r""" Evaluates the `quantum phase estimation algorithm <https://en.wikipedia.org/wiki/Quantum_phase_estimation_algorithm>`_. The unitary to estimate is expected to be given as Python function, which is called on ``args``. Parameters ---------- args : list A list of arguments (could be QuantumVariables) which represent the state, the quantum phase estimation is performed on. U : function A Python function, which will receive the list ``args`` as arguments in the course of this algorithm. precision : int, optional The precision of the estimation. The default is None. target : QuantumFloat, optional A target QuantumFloat to perform the estimation into. The default is None. If given neither a precision nor a target, an Exception will be raised. iter_spec : bool, optional If set to ``True``, ``U`` will be called with the additional keyword ``iter = i`` where ``i`` is the amount of iterations to perform (instead of simply calling ``U`` for ``i`` times). The default is False. ctrl_method : string, optional Allows to specify which method should be used to generate the controlled U circuit. For more information check :meth:`.control <qrisp.Operation.control>`. The default is None. kwargs : dict, optional A dictionary of keyword arguments to pass to ``U``. The default is {}. Raises ------ Exception Tried to perform quantum phase estimation without precision specification. Returns ------- res : QuantumFloat The QuantumFloat containing the estimated phase as a fraction of $2 \pi$. Examples -------- We define a function that applies two phase gates onto its input and estimate the applied phase. :: from qrisp import p, QuantumVariable, QPE, multi_measurement def U(qv): x = 0.5 y = 0.125 p(x*2*np.pi, qv[0]) p(y*2*np.pi, qv[1]) qv = QuantumVariable(2) h(qv) res = QPE(qv, U, precision = 3) >>> multi_measurement([qv, res]) {('00', 0.0): 0.25, ('10', 0.5): 0.25, ('01', 0.125): 0.25, ('11', 0.625): 0.25} >>> res.qs.depth() 66 During the phase estimation, ``U`` is called $2^{\text{precision}}$ times. We can reduce that number by abusing that we can bundle repeated calls into a single call with a modified phase. :: def U(qv, iter = None): x = 0.5 y = 0.125 p(x*2*np.pi*iter, qv[0]) p(y*2*np.pi*iter, qv[1]) qv = QuantumVariable(2) h(qv) res = QPE(qv, U, precision = 3, iter_spec = True) >>> multi_measurement([qv, res]) {('00', 0.0): 0.25, ('10', 0.5): 0.25, ('01', 0.125): 0.25, ('11', 0.625): 0.25} >>> res.qs.depth() 34 """ from qrisp import QuantumFloat, control if target is None: if precision is None: raise Exception( "Tried to perform quantum phase estimation without" "precision specification" ) qpe_res = QuantumFloat(precision, -precision, signed=False) else: qpe_res = target h(qpe_res) for i in range(qpe_res.size): if iter_spec: with control(qpe_res[i], ctrl_method=ctrl_method): U(args, iter=2**i, **kwargs) else: with control(qpe_res[i], ctrl_method=ctrl_method): for j in range(2**i): U(args, **kwargs) QFT(qpe_res, inv=True) return qpe_res
[docs]def quantum_counting(qv, oracle, precision): """ This algorithm estimates the amount of solutions for a given Grover oracle. Parameters ---------- qv : QuantumVariable The QuantumVariable on which to evaluate. oracle : function The oracle function. precision : int The precision to perform the quantum phase estimation with. Returns ------- M : float An estimate of the amount of solutions. Examples -------- We create an oracle, which performs a simple phase flip on the last qubit. :: from qrisp import quantum_counting, z, QuantumVariable def oracle(qv): z(qv[-1]) We expect half of the state-space of the input to be a solution. For 3 qubits, the state space is $2^3 = 8$ dimensional. >>> quantum_counting(QuantumVariable(3), oracle, 3) 3.999999999999999 For 4 qubits, the state space is $2^4 = 16$ dimensional. >>> quantum_counting(QuantumVariable(4), oracle, 3) 7.999999999999998 """ from qrisp import gate_wrap from qrisp.grover import diffuser @gate_wrap def grover_operator(qv): oracle(qv) diffuser(qv) h(qv) res = QPE(qv, grover_operator, precision=precision) mes_res = res.get_measurement() theta = min(list(mes_res.keys())[:1]) * 2 * np.pi N = 2**qv.size M = N * np.sin(theta / 2) ** 2 return M
[docs]def HHL(qv, hamiltonian_evolution, ev_inversion, precision): """ Evaluates the HHL algorithm. Parameters ---------- qv : TYPE DESCRIPTION. hamiltonian_evolution : TYPE DESCRIPTION. ev_inversion : TYPE DESCRIPTION. precision : TYPE DESCRIPTION. Returns ------- ancilla : TYPE DESCRIPTION. """ from qrisp import QuantumBool, invert # Perform quantum phase estimation res = QPE(qv, hamiltonian_evolution, precision) ancilla = QuantumBool() ev_inversion(res, ancilla) # Perform quantum phase estimation inverse with invert(): QPE(qv, hamiltonian_evolution, target=res) res.delete() return ancilla
def fredkin_qc(num_ctrl_qubits=1, ctrl_state=-1, method="gray"): from qrisp import QuantumCircuit, XGate mcx_gate = XGate().control().control(ctrl_state=ctrl_state, method=method) qc = QuantumCircuit(num_ctrl_qubits + 2) qc.cx(qc.qubits[-1], qc.qubits[-2]) qc.append(mcx_gate, qc.qubits) qc.cx(qc.qubits[-1], qc.qubits[-2]) return qc
[docs]def demux(input, ctrl_qv, output=None, ctrl_method=None, permit_mismatching_size=False, parallelize_qc = False): """ This functions allows moving an input value into an iterable output, where the position is specified by a ``QuantumFloat``. Demux is short for demultiplexer and is a standard component in `classical electrical circuitry <https://en.wikipedia.org/wiki/Multiplexer>`_. Demux can either move qubit states into a QuantumVariable or ``QuantumVariables`` into ``QuantumArrays``. This function can also be used to "in-place demux" the 0-th entry of an iterable to the position specified by ``ctrl_qv``. For more information on this, check the second example. Parameters ---------- input : Qubit or QuantumVariable The input value that is supposed to be moved. ctrl_qv : QuantumFloat The QuantumFloat specifying to which output the input should be moved. output : QuantumVariable or QuantumArray, optional The output object, where the input should end up. By default, a new object (QuantumVariable or QuantumArray) is created. Note that when this parameter is given, it is guaranteed, that the 0-th entry will be moved to the desired position, the other entries can also be permuted away from their original position. ctrl_method : string, optional The ``ctrl_method`` string passed to the :ref:`control environment <ControlEnvironment>` to generate controlled swaps. permit_mismatching_size : bool, optional If set to False, an exception will be raised, if the state-space dimension of `ctrl_qv`` is differing from the amount of outputs. The default is False. parallelize_qc : bool, optional If set to True, this option reduces (de)allocates additional qubits to reduce the depth. The default is False. Raises ------ Exception Tried to demux with mismatchingly sized control input. Returns ------- output : QuantumVariable or QuantumArray The output object with the input signal placed at the index specified by ``ctrl_qv``. Examples -------- We create a ``QuantumBool`` and demux it into a ``QuantumArray`` :: from qrisp import * qb = QuantumBool() qb.flip() index = QuantumFloat(2) h(index[1]) res_array = demux(qb, index) >>> print(multi_measurement([index, res_array])) {(0, OutcomeArray([1., 0., 0., 0.])): 0.5, (2, OutcomeArray([0., 0., 1., 0.])): 0.5} Demux can also be used to move the 0-th entry of a ``QuantumArray`` in-place. :: qa = QuantumArray(shape = 4, qtype = qb) qa[0].flip() demux(qa[0], index, qa) >>> print(multi_measurement([index, qa])) {(0, OutcomeArray([1., 0., 0., 0.])): 0.5, (2, OutcomeArray([0., 0., 1., 0.])): 0.5} For low-level manipulations, demux can move information within ``QuantumVariables``. :: qf = QuantumVariable(4) qf[:] = "1000" demux(qf[0], index, qf) >>> print(multi_measurement([index, qf])) {(0, '1000'): 0.5, (2, '0010'): 0.5} """ from qrisp import QuantumArray, QuantumVariable, Qubit, control, swap if output is None: if isinstance(input, QuantumVariable): output = QuantumArray(input, 2 ** len(ctrl_qv)) elif isinstance(input, Qubit): output = QuantumVariable(2 ** len(ctrl_qv)) else: raise Exception("Don't know how to handle input type " + str(type(input))) else: if isinstance(output, QuantumArray): for qv in output.flatten()[1:]: if qv.name == input.name: raise Exception( "Tried to in-place demux QuantumArray entry," "which is not a 0-th position" ) elif isinstance(output, QuantumVariable): for qb in output.reg[1:]: if qb.identifier == input.identifier: raise Exception( "Tried to in-place demux QuantumVariable entry," "which is not a 0-th position" ) n = int(np.ceil(np.log2(len(output)))) N = 2**n if len(output) != 2 ** len(ctrl_qv) and not permit_mismatching_size: raise Exception("Tried to demux with mismatching sized control input") if hash(input) != hash(output[0]): swap(input, output[0]) if not len(ctrl_qv): return output if len(output) > 2 ** (len(ctrl_qv) - 1): with control(ctrl_qv[-1], ctrl_method=ctrl_method): swap(output[0], output[N // 2]) else: demux( output[0], ctrl_qv[:-1], output, ctrl_method=ctrl_method, permit_mismatching_size=permit_mismatching_size, ) return output if n > 1: if parallelize_qc: demux_ancilla = QuantumVariable(len(ctrl_qv)-1) cx(ctrl_qv[:-1], demux_ancilla) ctrl_qubits = list(demux_ancilla) else: ctrl_qubits = ctrl_qv[:-1] demux( output[0], ctrl_qubits, #ctrl_qv[:-1], output[: N // 2], ctrl_method=ctrl_method, permit_mismatching_size=permit_mismatching_size, parallelize_qc=parallelize_qc ) demux( output[N // 2], ctrl_qv[:-1], output[N // 2 :], ctrl_method=ctrl_method, permit_mismatching_size=permit_mismatching_size, parallelize_qc=parallelize_qc ) if parallelize_qc: cx(ctrl_qv[:-1], demux_ancilla) demux_ancilla.delete() return output
def q_indexing(q_array, index): from qrisp import invert with invert(): demux(q_array[0], index, q_array, ctrl_method="gray_pt") res = q_array[0].duplicate(init=True) demux(q_array[0], index, q_array, ctrl_method="gray_pt") return res def q_swap_into(q_array, index, qv): from qrisp import invert, swap with invert(): demux(q_array[0], index, q_array, ctrl_method="gray_pt") swap(q_array[0], qv) demux(q_array[0], index, q_array, ctrl_method="gray_pt")
[docs]def cyclic_shift(iterable, shift_amount = 1): r""" Performs a cyclic shift of the values of an iterable with logarithmic depth. The shifting amount can be specified. Parameters ---------- iterable : list[Qubit] or list[QuantumVariable] or QuantumArray The iterable to be shifted. shift_amount : integer or QuantumFloat, optional The iterable will be shifted by that amount. The default is 1. Examples -------- We create a QuantumArray, initiate a sequence of increments and perform a cyclic shift. >>> from qrisp import QuantumFloat, QuantumArray, cyclic_shift >>> import numpy as np >>> qa = QuantumArray(QuantumFloat(3), 8) >>> qa[:] = np.arange(8) >>> cyclic_shift(qa, shift_amount = 2) >>> print(qa) {OutcomeArray([6, 7, 0, 1, 2, 3, 4, 5]): 1.0} We do something similar to demonstrate the shift by quantum values. For this we initiate a :ref:`QuantumFloat` in the superposition of 0, 1 and -3. >>> shift_amount = QuantumFloat(3, signed = True) >>> shift_amount[:] = {0 : 3**-0.5, 1: 3**-0.5, -3 : 3**-0.5} >>> qa = QuantumArray(QuantumFloat(3), 8) >>> qa[:] = np.arange(8) >>> cyclic_shift(qa, shift_amount) >>> print(qa) {OutcomeArray([0, 1, 2, 3, 4, 5, 6, 7]): 0.3333, OutcomeArray([7, 0, 1, 2, 3, 4, 5, 6]): 0.3333, OutcomeArray([3, 4, 5, 6, 7, 0, 1, 2]): 0.3333} """ from qrisp import QuantumFloat, control, QuantumBool, cx if isinstance(shift_amount, QuantumFloat): if shift_amount.mshape[0] < 0: raise Exception("Tried to quantum shift by non-integer QuantumFloat") if shift_amount.signed: with control(shift_amount.sign()): cyclic_shift(iterable, -2**(shift_amount.mshape[1])) for i in range(*shift_amount.mshape): with control(shift_amount.significant(i)): cyclic_shift(iterable, 2**i) return N = len(iterable) n = int(np.floor(np.log2(N))) if N == 0 or not shift_amount%N: return if shift_amount < 0: return cyclic_shift(iterable[::-1], -shift_amount) if shift_amount != 1: perm = np.arange(N) perm = (perm - shift_amount)%(N) permute_iterable(iterable, perm) return singular_shift(iterable[:2**n]) singular_shift([iterable[0]] + list(iterable[2**n:]), use_saeedi = True)
def singular_shift(iterable, use_saeedi = False): N = len(iterable) if N in [0,1]: return if use_saeedi: #Strategy from https://arxiv.org/abs/1304.7516 #Seems to perform worse when shifting by a quantum float #But better when the shift length is not a power of 2 for i in range(N//2): if (-i)%N == i+1 or i+1 >= N: continue swap(iterable[-i], iterable[i+1]) for i in range(N//2): if (-i)%N == i+2 or i+2 >= N: continue swap(iterable[-i], iterable[i+2]) else: correction_indices = [] for i in range(len(iterable)//2): swap_tuple = (2*i, 2*i+1) swap(iterable[swap_tuple[0]], iterable[swap_tuple[1]]) correction_indices.append(swap_tuple[0]) singular_shift([iterable[i] for i in correction_indices]) def to_cycles(perm): pi = {i: perm[i] for i in range(len(perm))} cycles = [] while pi: elem0 = next(iter(pi)) # arbitrary starting element this_elem = pi[elem0] next_item = pi[this_elem] cycle = [] while True: cycle.append(this_elem) del pi[this_elem] this_elem = next_item if next_item in pi: next_item = pi[next_item] else: break cycles.append(cycle) return cycles
[docs]def permute_iterable(iterable, perm): """ Applies an arbitrary permutation to an iterable with logarithmic depth. Parameters ---------- iterable : QuantumArray or List[QuantumVariable] or QuantumVariable or List[Qubit] The iterable to perform the permutation on. perm : List[integer] A list specifying the permutation. Examples -------- We create a QuantumArray containing increments and apply a specified permutation. >>> from qrisp import QuantumFloat, QuantumArray, permute_iterable >>> import numpy as np >>> qa = QuantumArray(QuantumFloat(3), 8) >>> qa[:] = np.arange(8) >>> permute_iterable(qa, perm = [1,0,3,7,5,2,6,4]) >>> print(qa) {OutcomeArray([1, 0, 3, 7, 5, 2, 6, 4]): 1.0} >>> print(qa.qs) :: QuantumCircuit: -------------- qa.0: ────────────X────────────────────── qa.1: ──────X─────┼────────────────────── │ │ qa.2: ──────┼──X──┼────────────────────── ┌───┐ │ │ │ qa_1.0: ┤ X ├─┼──┼──X────────────────────── └───┘ │ │ qa_1.1: ──────X──┼───────────────────────── qa_1.2: ─────────X───────────────────────── qa_2.0: ───────────────────────────X─────── ┌───┐ │ qa_2.1: ┤ X ├──────────────────────┼──X──── └───┘ │ │ qa_2.2: ───────────────────────────┼──┼──X─ ┌───┐ │ │ │ qa_3.0: ┤ X ├──────────X───────────┼──┼──┼─ ├───┤ │ │ │ │ qa_3.1: ┤ X ├──────────┼──X────────┼──┼──┼─ └───┘ │ │ │ │ │ qa_3.2: ───────────────┼──┼──X─────┼──┼──┼─ │ │ │ │ │ │ qa_4.0: ──────X────────┼──┼──┼─────┼──┼──┼─ │ │ │ │ │ │ │ qa_4.1: ──────┼──X─────┼──┼──┼─────┼──┼──┼─ ┌───┐ │ │ │ │ │ │ │ │ qa_4.2: ┤ X ├─┼──┼──X──┼──┼──┼─────┼──┼──┼─ ├───┤ │ │ │ │ │ │ │ │ │ qa_5.0: ┤ X ├─X──┼──┼──┼──┼──┼──X──X──┼──┼─ └───┘ │ │ │ │ │ │ │ │ qa_5.1: ─────────X──┼──┼──┼──┼──┼──X──X──┼─ ┌───┐ │ │ │ │ │ │ │ qa_5.2: ┤ X ├───────X──┼──┼──┼──┼──┼──X──X─ └───┘ │ │ │ │ │ │ qa_6.0: ───────────────┼──┼──┼──┼──┼──┼──── ┌───┐ │ │ │ │ │ │ qa_6.1: ┤ X ├──────────┼──┼──┼──┼──┼──┼──── ├───┤ │ │ │ │ │ │ qa_6.2: ┤ X ├──────────┼──┼──┼──┼──┼──┼──── ├───┤ │ │ │ │ │ │ qa_7.0: ┤ X ├──────────X──┼──┼──X──┼──┼──── ├───┤ │ │ │ │ qa_7.1: ┤ X ├─────────────X──┼─────X──┼──── ├───┤ │ │ qa_7.2: ┤ X ├────────────────X────────X──── └───┘ Live QuantumVariables: --------------------- QuantumFloat qa QuantumFloat qa_1 QuantumFloat qa_2 QuantumFloat qa_3 QuantumFloat qa_4 QuantumFloat qa_5 QuantumFloat qa_6 QuantumFloat qa_7 """ from sympy.combinatorics import Permutation inv_perm = list(Permutation(perm)**-1) cycles = to_cycles(inv_perm) for c in cycles: cyclic_shift([iterable[i] for i in c], 1)
[docs]def amplitude_amplification(args, state_function, oracle_function, kwargs_oracle={}, iter=1): r""" This method performs `quantum amplitude amplification <https://arxiv.org/abs/quant-ph/0005055>`_. The problem of quantum amplitude amplification is described as follows: * Given a unitary operator :math:`\mathcal{A}`, let :math:`\ket{\Psi}=\mathcal{A}\ket{0}`. * Write :math:`\ket{\Psi}=\ket{\Psi_1}+\ket{\Psi_0}` as a superposition of the orthogonal good and bad components of :math:`\ket{\Psi}`. * Enhance the probability :math:`a=\langle\Psi_1|\Psi_1\rangle` that a measurement of $\ket{\Psi}$ yields a good state. Let $\theta_a\in [0,\pi/2]$ such that $\sin^2(\theta_a)=a$. Then the amplitude amplification operator $\mathcal Q$ acts as .. math:: \mathcal Q^j\ket{\Psi}=\frac{1}{\sqrt{a}}\sin((2j+1)\theta_a)\ket{\Psi_1}+\frac{1}{\sqrt{1-a}}\cos((2j+1)\theta_a)\ket{\Psi_0}. Therefore, after $m$ iterations the probability of measuring a good state is $\sin^2((2m+1)\theta_a)$. Parameters ---------- args : QuantumVariable or list[QuantumVariable] The (list of) QuantumVariables which represent the state, the amplitude amplification is performed on. state_function : function A Python function preparing the state $\ket{\Psi}$. This function will receive the variables in the list ``args`` as arguments in the course of this algorithm. oracle_function : function A Python function tagging the good state $\ket{\Psi_1}$. This function will receive the variables in the list ``args`` as arguments in the course of this algorithm. kwargs_oracle : dict, optional A dictionary containing keyword arguments for the oracle. The default is {}. iter : int, optional The amount of amplitude amplification iterations to perform. The default is 1. Examples -------- We define a function that prepares the state :math:`\ket{\Psi}=\cos(\frac{\pi}{16})\ket{0}+\sin(\frac{\pi}{16})\ket{1}` and an oracle that tags the good state :math:`\ket{1}`. In this case, we have :math:`a=\sin^2(\frac{\pi}{16})\approx 0.19509`. :: from qrisp import z, ry, QuantumBool, amplitude_amplification import numpy as np def state_function(qb): ry(np.pi/8,qb) def oracle_function(qb): z(qb) qb = QuantumBool() state_function(qb) >>> qb.qs.statevector(decimals=5) 0.98079∣False⟩+0.19509∣True⟩ We can enhance the probability of measuring the good state with amplitude amplification: >>> amplitude_amplification([qb], state_function, oracle_function) >>> qb.qs.statevector(decimals=5) 0.83147*|False> + 0.55557*|True> >>> amplitude_amplification([qb], state_function, oracle_function) >>> qb.qs.statevector(decimals=5) 0.55557*|False> + 0.83147*|True> >>> amplitude_amplification([qb], state_function, oracle_function) >>> qb.qs.statevector(decimals=5) 0.19509*|False> + 0.98079*|True> """ from qrisp import merge, IterationEnvironment from qrisp.grover import diffuser merge(args) qs = recursive_qs_search(args)[0] with IterationEnvironment(qs, iter): oracle_function(*args, **kwargs_oracle) diffuser(args, state_function=state_function)
[docs]def QAE(args, state_function, oracle_function, kwargs_oracle={}, precision=None, target=None): r""" This method implements the canonical quantum amplitude estimation (QAE) algorithm by `Brassard et al. <https://arxiv.org/abs/quant-ph/0005055>`_. The problem of quantum amplitude estimation is described as follows: * Given a unitary operator :math:`\mathcal{A}`, let :math:`\ket{\Psi}=\mathcal{A}\ket{0}`. * Write :math:`\ket{\Psi}=\ket{\Psi_1}+\ket{\Psi_0}` as a superposition of the orthogonal good and bad components of :math:`\ket{\Psi}`. * Find an estimate for :math:`a=\langle\Psi_1|\Psi_1\rangle`, the probability that a measurement of $\ket{\Psi}$ yields a good state. Parameters ---------- args : QuantumVariable or list[QuantumVariable] The (list of) QuantumVariables which represent the state, the quantum amplitude estimation is performed on. state_function : function A Python function preparing the state :math:`\ket{\Psi}`. This function will receive the variables in the list ``args`` as arguments in the course of this algorithm. oracle_function : function A Python function tagging the good state :math:`\ket{\Psi_1}`. This function will receive the variables in the list ``args`` as arguments in the course of this algorithm. kwargs_oracle : dict, optional A dictionary containing keyword arguments for the oracle. The default is {}. precision : int, optional The precision of the estimation. The default is None. target : QuantumFloat, optional A target QuantumFloat to perform the estimation into. The default is None. If given neither a precision nor a target, an Exception will be raised. Returns ------- res : QuantumFloat A QuantumFloat encoding the angle :math:`\theta` as a fraction of :math:`\pi`, such that :math:`\tilde{a}=\sin^2(\theta)` is an estimate for :math:`a`. More precisely, we have :math:`\theta=\pi\frac{y}{M}` for :math:`y\in\{0,\dotsc,M-1\}` and :math:`M=2^{\text{precision}}`. After measurement, the estimate :math:`\tilde{a}=\sin^2(\theta)` satisfies .. math:: |a-\tilde{a}|\leq\frac{2\pi}{M}+\frac{\pi^2}{M^2} with probability of at least :math:`8/\pi^2`. Examples -------- We define a function that prepares the state :math:`\ket{\Psi}=\cos(\frac{\pi}{8})\ket{0}+\sin(\frac{\pi}{8})\ket{1}` and an oracle that tags the good state :math:`\ket{1}`. In this case, we have :math:`a=\sin^2(\frac{\pi}{8})`. :: from qrisp import z, ry, QuantumBool, QAE import numpy as np def state_function(qb): ry(np.pi/4,qb) def oracle_function(qb): z(qb) qb = QuantumBool() res = QAE([qb], state_function, oracle_function, precision=3) >>> res.get_measurement() {0.125: 0.5, 0.875: 0.5} That is, after measurement we find $\theta=\frac{\pi}{8}$ or $\theta=\frac{7\pi}{8}$ with probability $\frac12$, respectively. Therefore, we obtain the estimate $\tilde{a}=\sin^2(\frac{\pi}{8})$ or $\tilde{a}=\sin^2(\frac{7\pi}{8})$. In this case, both results coincide with the exact value $a$. **Numerical integration** Here, we demonstarate how to use QAE for numerical integration. Consider a continuous function $f\colon[0,1]\rightarrow[0,1]$. We wish to evaluate .. math:: A=\int_0^1f(x)\mathrm dx. For this, we set up the corresponding ``state_function`` acting on the ``input_list``: :: from qrisp import QuantumFloat, QuantumBool, control, z, h, ry, QAE import numpy as np n = 6 inp = QuantumFloat(n,-n) tar = QuantumBool() input_list = [inp, tar] Here, $N=2^n$ is the number of sampling points the function $f$ is evaluated on. The ``state_function`` acts as .. math:: \ket{0}\ket{0}\rightarrow\frac{1}{\sqrt{N}}\sum\limits_{x=0}^{N-1}\ket{x}\left(\sqrt{1-f(x/N)}\ket{0}+\sqrt{f(x/N)}\ket{1}\right). Then the probability of measuring $1$ in the target state ``tar`` is .. math:: p(1)=\frac{1}{N}\sum\limits_{x=0}^{N-1}f(x/N), which acts as an approximation for the value of the integral $A$. The ``oracle_function``, therefore, tags the $\ket{1}$ state of the target state: :: def oracle_function(inp, tar): z(tar) For example, if $f(x)=\sin^2(x)$ the ``state_function`` can be implemented as follows: :: def state_function(inp, tar): h(inp) N = 2**inp.size for k in range(inp.size): with control(inp[k]): ry(2**(k+1)/N,tar) Finally, we apply QAE and obtain an estimate $a$ for the value of the integral $A=0.27268$. :: prec = 6 res = QAE(input_list, state_function, oracle_function, precision=prec) meas_res = res.get_measurement() theta = np.pi*max(meas_res, key=meas_res.get) a = np.sin(theta)**2 >>> a 0.26430 """ state_function(*args) res = QPE(args, amplitude_amplification, precision, target, iter_spec=True, kwargs={'state_function':state_function, 'oracle_function':oracle_function, 'kwargs_oracle':kwargs_oracle}) return res